Probability that player wins the game (Markov chains)

expected valuegamblingmarkov chainsprobability

Players A and B take turns in answering trivia questions, starting with player A answering the first question. Each time A answers, she has a probability p1 of getting it right. Each time B plays, he has probability p2 of getting it right. Suppose that first player to answer correctly wins the game. What is the probability that A wins?

Attempt 1: I approached it with Markov chains with states $(A, B, win)$ representing A playing, B playing, and A winning the game respectively. If (pA, pB) denote the probability that A and B wins the game respectively, then I have the set of equations
$$pA = p1 + (1-p1)pB$$ $$pB = (1-p2)pA + p2 $$
The first equation says that if player A plays, she has a probability $p1$ of winning the game and $(1-p1)$ of getting the answer wrong, in which case it is player B's turn. The second equation says that player B has $p2$ chance of winning the game and with probability $(1-p2)$ it is player A's turn. However, the set of equations give the answer $pA=1$ which doesn't make sense.

Attempt 2: I augmented the states with $(A,B, Awins, Bwins)$, giving me the set of equations
$$pA = p1 + (1-p1)pB $$
$$pB = (1-p2)pA + p2*0 $$
The solution to this system is $pA = \frac{p1}{1-(1-p1)(1-p2)}$, which makes sense.

My questions are as follows:

  1. Where is the flaw in attempt1?
  2. How do we set up the states in such Markov chain type problems?
  3. How do we solve this problem in recursive fashion without the knowledge of Markov chains?

Thanks a ton!

Best Answer

I can think of two reasons why Attempt $1$ fails. 1) By setting up this mutually recursive relationship you're making the assumption that $pA$ and $pB$ aren't influenced by the choice of starting player. 2) We must have $pB=1-pA$ since the probability this game continues indefinitely is $0$. Therefore $pB$ can only depend on $p_1$ since $pA$ does not depend on $p_2$. But clearly we know from the problem statement that $pB$ should depend on $p_2$.

Here is how to solve this problem without Markov chains. Assuming player $A$ goes first, the probability $A$ wins within the first $n$ guess is given by $P(A_n)$. Then, $$P(A_n)=p_1+(1-p_1)(1-p_2)p_1+\cdots+(1-p_1)^{n-1}(1-p_2)^{n-1}p_1$$ E.g. to win on the second guess $A$ must mess up the first guess, have $B$ mess up their guess, and then guess it correctly. $$P(A)=\lim_{n\rightarrow \infty}P(A_n)=\dfrac{p_1}{1-(1-p_1)(1-p_2)}$$ by convergence of the geometric series since $(1-p_1)(1-p_2)<1$.

Related Question